2020 AMC 12A Problems/Problem 13
Problem
There are integers and each greater than such that
Solution
can be simplified to
The equation is then which implies that a2\frac{25}{36}>\frac{1}{2}b3frac{2}{3}frac{25}{36}c6frac{25}{36}a, b,cb\boxed{\textbf{(B) } 3.}$~lopkiloinm